Which one of the following, if substituted for the condition that if M is published in the fall, N must be published ...

Serra on May 12, 2019

Best approach to this question?

Could someone please review my approach to come to answer choice (B)? I got the answer correct but I was not able to finish this question before the time was up for the LG section. I did not deduce that Rule 4 was only relevant/impacting when the KN pair is in fall in my initial Game Setup, and then wasted several minutes looking at the Scenario when the KN pair is in the spring. Other than identifying this missed deduction in the game setup is there any other way to come to this answer choice more quickly? Or might there be a more efficient way to approach the problem? Please let me know! Thank you! STEP 1- We know that Rule 4 only has an impact when the KN pair is in the fall: Rule 4 tells us: M fall > N spring not N spring > not M fall We have the following Scenario when applying Rule 4 where it is relevant: fall: P K N O spring: M *L is an unrestricted variable and can be in either fall or spring. STEP 2- If we delete the impact of Rule 4 we have the following scenario: fall: M/P K N O spring: P/M *L is an unrestricted variable and can be in either fall or spring. STEP 3- What rule would make it necessary for P to be in the fall and M to be in the spring? Answer choice (B) tells us “If N is in the fall, P must also be in the fall.” If we applied this rule we have the following scenario: fall: P K N O spring: M *L is an unrestricted variable and can be in either fall or spring. As we can see above (B) is the correct answer choice.

Replies
Create a free account to read and take part in forum discussions.

Already have an account? log in

Serra on May 12, 2019

Please disregard my prior post as it contains a mistake. Sorry! Here is my proper question:

Could someone please list the inferences/deductions for this Game Setup beyond what is explicitly stated in the rules or as shown below? Just want to make sure that I am getting all of the deductions up front to speed up the time needed to answer the questions that follow. Thank you!

Deductions:
(1) L is the least restricted variable.
(2) The NK pair is the most restricted variable.
(3) Rule 4 is only relevant/impacting if the NK pair is in the fall.

SCENARIO 1: KN pair is published in the fall

fall: K N O P
spring: M

*L is an unrstricted variable and can be in either fall or spring

SCENARIO 2: KN pair is published in the spring

fall: P/M
spring: M/P K N

*O and L are unrestricted variables and can be in either fall or spring

Serra on May 12, 2019

Could someone please review my approach to come to answer choice (B)? I got the answer correct but I was not able to finish this question before the time was up for the LG section. I did not deduce that Rule 4 was only relevant/impacting when the KN pair is in fall in my initial Game Setup, and then wasted several minutes looking at the Scenario when the KN pair is in the spring. Other than identifying this missed deduction in the game setup is there any other way to come to this answer choice more quickly? Or might there be a more efficient way to approach the problem? Please let me know! Thank you!

STEP 1- We know that Rule 4 only has an impact when the KN pair is in the fall:

Rule 4 tells us:
M fall > N spring
not N spring > not M fall

We have the following Scenario when applying Rule 4 where it is relevant:

fall: P K N O
spring: M

*L is an unrestricted variable and can be in either fall or spring.

STEP 2- If we delete the impact of Rule 4 we have the following scenario:

fall: M/P K N O
spring: P/M

*L is an unrestricted variable and can be in either fall or spring.

STEP 3- What rule would make it necessary for P to be in the fall and M to be in the spring?

Answer choice (B) tells us “If N is in the fall, P must also be in the fall.” If we applied this rule we have the following scenario:

fall: P K N O
spring: M

*L is an unrestricted variable and can be in either fall or spring.

As we can see above (B) is the correct answer choice.

Ravi on May 13, 2019

@msaber,

Great question.

The deductions you made from the rules in the setup are excellent, and
you did not leave anything out. With those two scenarios, you are
well-equipped to answer the questions in this game quickly.

Does this make sense? Let us know if you have any other questions!

Serra on May 15, 2019

Great thank you!